Distance between a vector and a subspace












1














Let $V$ be a vector space of finite dimension, and let $0 neq u in V$. Denote $U=(span text{{u}})^bot$.



Prove that for every $v in V$



$$dist(v,U)=frac {lvert leftlangle v,u rightranglervert}{lVert urVert}$$










share|cite|improve this question




















  • 1




    Think in terms of projections.
    – Anurag A
    Jun 10 '17 at 17:07
















1














Let $V$ be a vector space of finite dimension, and let $0 neq u in V$. Denote $U=(span text{{u}})^bot$.



Prove that for every $v in V$



$$dist(v,U)=frac {lvert leftlangle v,u rightranglervert}{lVert urVert}$$










share|cite|improve this question




















  • 1




    Think in terms of projections.
    – Anurag A
    Jun 10 '17 at 17:07














1












1








1


1





Let $V$ be a vector space of finite dimension, and let $0 neq u in V$. Denote $U=(span text{{u}})^bot$.



Prove that for every $v in V$



$$dist(v,U)=frac {lvert leftlangle v,u rightranglervert}{lVert urVert}$$










share|cite|improve this question















Let $V$ be a vector space of finite dimension, and let $0 neq u in V$. Denote $U=(span text{{u}})^bot$.



Prove that for every $v in V$



$$dist(v,U)=frac {lvert leftlangle v,u rightranglervert}{lVert urVert}$$







linear-algebra vector-spaces inner-product-space orthogonality






share|cite|improve this question















share|cite|improve this question













share|cite|improve this question




share|cite|improve this question








edited Nov 20 '18 at 11:49









José Carlos Santos

150k22122222




150k22122222










asked Jun 10 '17 at 17:01









user401516

91539




91539








  • 1




    Think in terms of projections.
    – Anurag A
    Jun 10 '17 at 17:07














  • 1




    Think in terms of projections.
    – Anurag A
    Jun 10 '17 at 17:07








1




1




Think in terms of projections.
– Anurag A
Jun 10 '17 at 17:07




Think in terms of projections.
– Anurag A
Jun 10 '17 at 17:07










1 Answer
1






active

oldest

votes


















2














The distance form $v$ to $U$ is the distance from $v$ to the only $win U$ such that $v-w$ is orthogonal to $U$, that is, such that $v-w=lambda u$, for some $lambdainmathbb R$. Since $v-frac{langle v,urangle}{|u|^2}uin U$, you just have to take $lambda=frac{langle v,urangle}{|u|^2}$. So, $w=v-lambda u=v-frac{langle v,urangle}{|u|^2}u$ and the distance from $v$ to $w$ is equal to $frac{bigl|langle v,uranglebigr|}{|u|}$.






share|cite|improve this answer





















    Your Answer





    StackExchange.ifUsing("editor", function () {
    return StackExchange.using("mathjaxEditing", function () {
    StackExchange.MarkdownEditor.creationCallbacks.add(function (editor, postfix) {
    StackExchange.mathjaxEditing.prepareWmdForMathJax(editor, postfix, [["$", "$"], ["\\(","\\)"]]);
    });
    });
    }, "mathjax-editing");

    StackExchange.ready(function() {
    var channelOptions = {
    tags: "".split(" "),
    id: "69"
    };
    initTagRenderer("".split(" "), "".split(" "), channelOptions);

    StackExchange.using("externalEditor", function() {
    // Have to fire editor after snippets, if snippets enabled
    if (StackExchange.settings.snippets.snippetsEnabled) {
    StackExchange.using("snippets", function() {
    createEditor();
    });
    }
    else {
    createEditor();
    }
    });

    function createEditor() {
    StackExchange.prepareEditor({
    heartbeatType: 'answer',
    autoActivateHeartbeat: false,
    convertImagesToLinks: true,
    noModals: true,
    showLowRepImageUploadWarning: true,
    reputationToPostImages: 10,
    bindNavPrevention: true,
    postfix: "",
    imageUploader: {
    brandingHtml: "Powered by u003ca class="icon-imgur-white" href="https://imgur.com/"u003eu003c/au003e",
    contentPolicyHtml: "User contributions licensed under u003ca href="https://creativecommons.org/licenses/by-sa/3.0/"u003ecc by-sa 3.0 with attribution requiredu003c/au003e u003ca href="https://stackoverflow.com/legal/content-policy"u003e(content policy)u003c/au003e",
    allowUrls: true
    },
    noCode: true, onDemand: true,
    discardSelector: ".discard-answer"
    ,immediatelyShowMarkdownHelp:true
    });


    }
    });














    draft saved

    draft discarded


















    StackExchange.ready(
    function () {
    StackExchange.openid.initPostLogin('.new-post-login', 'https%3a%2f%2fmath.stackexchange.com%2fquestions%2f2317509%2fdistance-between-a-vector-and-a-subspace%23new-answer', 'question_page');
    }
    );

    Post as a guest















    Required, but never shown

























    1 Answer
    1






    active

    oldest

    votes








    1 Answer
    1






    active

    oldest

    votes









    active

    oldest

    votes






    active

    oldest

    votes









    2














    The distance form $v$ to $U$ is the distance from $v$ to the only $win U$ such that $v-w$ is orthogonal to $U$, that is, such that $v-w=lambda u$, for some $lambdainmathbb R$. Since $v-frac{langle v,urangle}{|u|^2}uin U$, you just have to take $lambda=frac{langle v,urangle}{|u|^2}$. So, $w=v-lambda u=v-frac{langle v,urangle}{|u|^2}u$ and the distance from $v$ to $w$ is equal to $frac{bigl|langle v,uranglebigr|}{|u|}$.






    share|cite|improve this answer


























      2














      The distance form $v$ to $U$ is the distance from $v$ to the only $win U$ such that $v-w$ is orthogonal to $U$, that is, such that $v-w=lambda u$, for some $lambdainmathbb R$. Since $v-frac{langle v,urangle}{|u|^2}uin U$, you just have to take $lambda=frac{langle v,urangle}{|u|^2}$. So, $w=v-lambda u=v-frac{langle v,urangle}{|u|^2}u$ and the distance from $v$ to $w$ is equal to $frac{bigl|langle v,uranglebigr|}{|u|}$.






      share|cite|improve this answer
























        2












        2








        2






        The distance form $v$ to $U$ is the distance from $v$ to the only $win U$ such that $v-w$ is orthogonal to $U$, that is, such that $v-w=lambda u$, for some $lambdainmathbb R$. Since $v-frac{langle v,urangle}{|u|^2}uin U$, you just have to take $lambda=frac{langle v,urangle}{|u|^2}$. So, $w=v-lambda u=v-frac{langle v,urangle}{|u|^2}u$ and the distance from $v$ to $w$ is equal to $frac{bigl|langle v,uranglebigr|}{|u|}$.






        share|cite|improve this answer












        The distance form $v$ to $U$ is the distance from $v$ to the only $win U$ such that $v-w$ is orthogonal to $U$, that is, such that $v-w=lambda u$, for some $lambdainmathbb R$. Since $v-frac{langle v,urangle}{|u|^2}uin U$, you just have to take $lambda=frac{langle v,urangle}{|u|^2}$. So, $w=v-lambda u=v-frac{langle v,urangle}{|u|^2}u$ and the distance from $v$ to $w$ is equal to $frac{bigl|langle v,uranglebigr|}{|u|}$.







        share|cite|improve this answer












        share|cite|improve this answer



        share|cite|improve this answer










        answered Jun 10 '17 at 17:13









        José Carlos Santos

        150k22122222




        150k22122222






























            draft saved

            draft discarded




















































            Thanks for contributing an answer to Mathematics Stack Exchange!


            • Please be sure to answer the question. Provide details and share your research!

            But avoid



            • Asking for help, clarification, or responding to other answers.

            • Making statements based on opinion; back them up with references or personal experience.


            Use MathJax to format equations. MathJax reference.


            To learn more, see our tips on writing great answers.





            Some of your past answers have not been well-received, and you're in danger of being blocked from answering.


            Please pay close attention to the following guidance:


            • Please be sure to answer the question. Provide details and share your research!

            But avoid



            • Asking for help, clarification, or responding to other answers.

            • Making statements based on opinion; back them up with references or personal experience.


            To learn more, see our tips on writing great answers.




            draft saved


            draft discarded














            StackExchange.ready(
            function () {
            StackExchange.openid.initPostLogin('.new-post-login', 'https%3a%2f%2fmath.stackexchange.com%2fquestions%2f2317509%2fdistance-between-a-vector-and-a-subspace%23new-answer', 'question_page');
            }
            );

            Post as a guest















            Required, but never shown





















































            Required, but never shown














            Required, but never shown












            Required, but never shown







            Required, but never shown

































            Required, but never shown














            Required, but never shown












            Required, but never shown







            Required, but never shown







            Popular posts from this blog

            Can a sorcerer learn a 5th-level spell early by creating spell slots using the Font of Magic feature?

            Does disintegrating a polymorphed enemy still kill it after the 2018 errata?

            A Topological Invariant for $pi_3(U(n))$